Find c likes the series is convergent

  • Thread starter Thread starter marrie
  • Start date Start date
  • Tags Tags
    Convergent Series
Click For Summary
To determine the value of c for which the integral ∫₀^∞ (2x/(x²+1) - c/(2x+1)) dx converges, it is essential to analyze the convergence of each term separately. The discussion suggests that the denominator must grow faster than the numerator for convergence. Participants emphasize the importance of finding a common denominator to simplify the expression. The hints provided guide towards understanding how to balance the growth rates of the numerator and denominator. Ultimately, the goal is to identify the correct value of c that ensures the integral converges.
marrie
Messages
2
Reaction score
0
Find c ∈ IR, like ∫∞ ( 2x - c ) dx is convergent
0 X^2 +1 2x+1

I need your help because I was trying to resolve the problem, but I couldn't, is difficult for me.
Please help me!
 
Physics news on Phys.org
Welcome to PF!

Hi marrie! Welcome to PF! :smile:
marrie said:
Find c ∈ IR, like ∫∞ ( 2x - c ) dx is convergent
0 X^2 +1 2x+1

hmm :rolleyes: … you mean \int_0^{\infty}\left(\frac{2x}{x^2+1} - \frac{c}{2x+1}\right)dx :wink:

Hint:

i] can you integrate each of them separately? does each converge or diverge?

ii] how much faster do you think the bottom needs to increase than the top for the integral to converge? :smile:
 
Ok, if I integrate each of them separately, I believe that the series diverge.
And I know that the bottom needs to increase than for the top for the integral converge.
thanks for your hints!
 
Hi marrie! :smile:
marrie said:
Ok, if I integrate each of them separately, I believe that the series diverge.
And I know that the bottom needs to increase than for the top for the integral converge.

ah … but how much more than the top? :wink:

anyway, write the whole thing over a common denominator …

then what does c have to be to make the top increase slowly enough? :smile:
 
Question: A clock's minute hand has length 4 and its hour hand has length 3. What is the distance between the tips at the moment when it is increasing most rapidly?(Putnam Exam Question) Answer: Making assumption that both the hands moves at constant angular velocities, the answer is ## \sqrt{7} .## But don't you think this assumption is somewhat doubtful and wrong?

Similar threads

  • · Replies 7 ·
Replies
7
Views
2K
  • · Replies 3 ·
Replies
3
Views
1K
  • · Replies 2 ·
Replies
2
Views
1K
  • · Replies 8 ·
Replies
8
Views
2K
  • · Replies 2 ·
Replies
2
Views
2K
  • · Replies 2 ·
Replies
2
Views
2K
Replies
7
Views
2K
  • · Replies 5 ·
Replies
5
Views
2K
  • · Replies 48 ·
2
Replies
48
Views
5K
  • · Replies 6 ·
Replies
6
Views
2K